K
Khách

Hãy nhập câu hỏi của bạn vào đây, nếu là tài khoản VIP, bạn sẽ được ưu tiên trả lời.

10 tháng 2 2019

Ta có:

Giải bài 6 trang 106 SGK Đại Số 10 | Giải toán lớp 10

Áp dụng bất đẳng thức Cô-si cho hai số dương

Giải bài 6 trang 106 SGK Đại Số 10 | Giải toán lớp 10

Dấu "=" xảy ra khi và chỉ khi a = b = c

24 tháng 11 2015

ta cần chứng minh nó lớn hơn 1 và nhỏ hơn 2

Do a;b;c và d là các số nguyên dương => 
a + b + c < a + b + c + d 
a + b + d < a + b + c + d 
a + c + d < a + b + c + d 
b + c + d < a + b + c + d 
=> a/(a + b + c) > a/(a + b + c + d) (1) 
b/(a + b + d) > b/(a + b + c + d) (2) 
c/(b + c + d) > c/(a + b + c + d) (3) 
d/(a + c + d) > d/(a + b + c + d) (4) 
Từ (1);(2);(3) và (4) 
=> a/(a + b + c) + b/(a + b + d) + c/(b + c + d) + d/(a + c + d) > a/(a + b + c + d) + b/(a + b + c + d) + c/(a + b + c + d) + d/(a + b + c + d) 
=> a/(a + b + c) + b/(a + b + d) + c/(b + c + d) + d/(a + c + d) > (a + b + c + d)/(a + b + c + d) 
=> a/(a + b + c) + b/(a + b + d) + c/(b + c + d) + d/(a + c + d) > 1 
=> B > 1 (*) 

Ta có: (a + b + c)(a + d) - a(a + b + c + d) 
= a² + ad + ab + bd + ac + cd - (a² + ab + ac + ad) 
= a² + ad + ab + bd + ac + cd - a² - ab - ac - ad 
= bd + cd 
Do a;b;c và d là số nguyên dương 
=> bd + cd > 0 
=> (a + b + c)(a + d) - a(a + b + c + d) > 0 
=> (a + b + c)(a + d) > a(a + b + c + d) 
=> (a + d)/(a + b + c + d) > a/(a + b + c) (5) 
Chứng minh tương tự ta được: 
(b + c)/(a + b + c + d) > b/(a + b + d) (6) 
(a + c)/(a + b + c + d) > c/(b + c + d) (7) 
(b + d)/(a + b + c + d) > d/(a + c + d) (8) 
Cộng vế với vế của (5);(6);(7) và (8) ta được: 
(a + d)/(a + b + c + d) + (b + c)/(a + b + c + d) + (a + c)/(a + b + c + d) + (b + d)/(a + b + c + d) > a/(a + b + c) + b/(a + b + d) + c/(b + c + d) + d/(a + c + d) 
=> (a + d + b + c + a + c + b + d)/(a + b + c + d) > B 
=> 2(a + b + c + d)/(a + b + c + d) > B 
=> 2 > B (*)(*) 
Từ (*) và (*)(*) 
=> 1 < B < 2 
=> B không phải là số nguyên

13 tháng 4 2016

Ta có: a/a+b <a/a+b+c    (1)

           b/b+c <b/a+b+c     (2) 

           c/c+a <c/a+b+c      (3)

Từ (1),(2),(3)  =>    a/a+b    +   b/b+c   +    c/c+a    >     a/a+b+c  +   b/a+b+c   +    c/a+b+c

                                                                                       = a+b+c/a+b+c

                                                                                       =1

VẬY : M>1

Ta có :

              a/a+b    <   a+c/a+b+c     (1)

              b/b+c    <   b+a/a+b+c     (2)

              c/c+a     <   c+b/a+b+c     (3)

Từ (1),(2),(3) =>  a/a+b    +   b/b+c   +    c/c+a    <     a+c/a+b+c    +      b+a/a+b+c      +    c+a/a+b+c 

                                                                                   =     2.(a+b+c)/a+b+c

                                                                                   =     2

=>          1<M<2          

=>          M không phải là số nguyên

8 tháng 3 2016

  ta co(:a+b)/c+(b+c)/a+(a+c)/b=a/c+c/a+a/b+b... 
theo bdt cauchy,ta co 
a/c+c/a>=2 
b/c+c/b>=2 
a/b+b/a>=2 
vay a/c+a/b+b/a+b/c+c/a+c/b>=6(dpcm) 
dau "="say ra khi a=b=c=1

12 tháng 3 2022

Câu hỏi của Adminbird - Toán lớp 7 - Học toán với OnlineMath

12 tháng 2 2022

Giúp mình câu này với ah.

 

M=a/a+b+b/b+c+c/c+a vs a,b,c lớn hơn 0

M=1+b+1+c+1+a=3+a,b,c

M là số nguyên

16 tháng 2 2016

Ta có a/b+c+b/a+c+c/a+b > a/a+b+c+b/b+c+a+c/b+c+a=a+b+c/a+b+c=1

=>M>1

Lại có M=(1-b/a+b)+(1- c/b+c)+(1-c/a+c)<3-(b/a+b+c+c/b+c+a+a/c+a+b)=3-1=2

=>M < 2

 do đo 1<M<2=>đpcm

2 tháng 7 2021

\(\dfrac{a}{b+2c}+\dfrac{b}{c+2a}+\dfrac{c}{a+2b}=\dfrac{a^2}{ab+2ac}+\dfrac{b^2}{bc+2ab}+\dfrac{c^2}{ac+2bc}\)

áp dụng BDT CAUCHY SCHAWRZ

\(=>\dfrac{a^2}{ab+2ac}+\dfrac{b^2}{bc+2ab}+\dfrac{c^2}{ac+2bc}\ge\dfrac{\left(a+b+c\right)^2}{ab+bc+ac+2ac+2ab+2bc}\)

\(=\dfrac{\left(a+b+c\right)^2}{3\left(ab+bc+ac\right)}\ge\dfrac{3\left(ab+bc+ac\right)}{3\left(ab+bc+ac\right)}=1\)

5 tháng 7 2021

cái chỗ bđt cauchy là bđt gì bạn có thể ghi cụ thể nó ra được ko ạ 

 

16 tháng 5 2022

Xét : \(\left(a^2+b^2+c^2+d^2\right)-\left(a+b+c+d\right)\)

 

        \(=a\left(a-1\right)+b\left(b-1\right)+c\left(c-1\right)+d\left(d-1\right)\)

 

Vì \(a\) là  số nguyên dương nên \(a,\left(a-1\right)\) là hai số tự nhiên liên tiếp . 

 

\(\Rightarrow a\left(a-1\right)\) chia hết cho 2. Tương tự ta có : \(b\left(b-1\right);c\left(c-1\right);d\left(d-1\right)\) đều chia hết cho 2.

 

\(\Rightarrow a\left(a-1\right)+b\left(b-1\right)+c\left(c-1\right)+d\left(d-1\right)\) là số chẵn . 

 

Lại có : \(a^2+c^2=b^2+d^2\Rightarrow a^2+b^2+c^2+d^2=2\left(b^2+d^2\right)\) là số chẵn .

 

Do đó : \(a+b+c+d\) là số chẵn mà \(a+b+c+d>2\) (Do \(a,b,c,d\inℕ^∗\))

 

Vậy : \(a+b+c+d\) là hợp số .

29 tháng 3

Xét : (�2+�2+�2+�2)−(�+�+�+�)

        =�(�−1)+�(�−1)+�(�−1)+�(�−1)

Vì  là  số nguyên dương nên �,(�−1) là hai số tự nhiên liên tiếp . 

⇒�(�−1) chia hết cho 2. Tương tự ta có : �(�−1);�(�−1);�(�−1) đều chia hết cho 2.

⇒�(�−1)+�(�−1)+�(�−1)+�(�−1) là số chẵn . 

Lại có : �2+�2=�2+�2⇒�2+�2+�2+�2=2(�2+�2) là số chẵn .

Do đó : �+�+�+� là số chẵn mà �+�+�+�>2 (Do �,�,�,�∈N∗)

Vậy : �+�+�+� là hợp số .

18 tháng 1 2017

Cho x,y,z là các số nguyên tố khác 2 và các số thực a,b,c thỏa mãn dãy tỉ số bằng nhau a-b/x=b-c/y=a-c/z.CMR a=b=c

Dễ thế mà chẳng ai làm được..